Show that a transformation is canonical

In summary, the conversation discusses how to show that a given transformation is canonical. The necessary conditions for a canonical transformation are [Q,P]_{q,p}=1, [Q,Q]_{q,p}=0, and [P,P]_{q,p}=0, where [f,g]_{p,q} represents the Poisson brackets of f and g with respect to p and q. The conversation also includes a calculation of these conditions for the given transformation, and a discussion of the mistakes made during the calculation. Eventually, the mistake is found and the results are found to be correct, concluding that the transformation is canonical.
  • #1
fluidistic
Gold Member
3,948
263

Homework Statement


Show that the following transformation is canonical:
[itex]Q=\ln \left ( \frac{\sin p }{q} \right )[/itex], [itex]P=q \cot p[/itex].


Homework Equations


A transformation is canonical if [itex]\dot Q=\frac{\partial H'}{\partial P}[/itex] and [itex]\dot P =-\frac{\partial H'}{\partial Q}[/itex].
H' is the Hamiltonian in function of Q and P.

The Attempt at a Solution


I've calculated [itex]\dot Q = \dot p \cot p - \frac{\dot q \sin p }{q}[/itex] and [itex]\dot P=\dot q \cot p - \frac{q \dot p}{\sin ^2 p}[/itex].
Now I guess I must check out if the conditions about the partial derivatives of the Hamiltonian are satisfied but I do not know either H(q,p) nor do I know H'(Q,P). I'm stuck here.
 
Physics news on Phys.org
  • #2
Ok guys, discard my attempt in my last post.
I've found in some class notes and in Landau&Lifgarbagez's book the necessary condition for a transformation to be canonical. If I'm not wrong, I must show that
I)[itex][Q,P]_{q,p}=1[/itex]
II)[itex][Q,Q]_{q,p}=0[/itex]
III)[itex][P,P]_{q,p}=0[/itex].
Where [itex][f,g]_{p,q}[/itex] denotes the Poisson brackets of f and g with respect to p and q. In other words, this is worth [itex]\frac{\partial f}{\partial p} \frac{\partial g}{\partial q}-\frac{\partial f}{\partial q} \frac{\partial g}{\partial p}[/itex].
Now I have that [itex]Q=\ln \left ( \frac{\sin p }{q} \right )[/itex] and [itex]P=q \cot p[/itex].
I tried to be careful in doing the partial derivatives. I found out that:
(1)[itex]\frac{\partial Q}{\partial p}=q \cot p[/itex]
(2)[itex]\frac{\partial P}{\partial q}= \cot p[/itex]
(3)[itex]\frac{\partial Q}{\partial q}=-\frac{1}{q}[/itex]
(4)[itex]\frac{\partial P}{\partial p}=q(-1- \cot ^2 p)[/itex]

Using this, I found out I) to be worth [itex](1-q) \cot ^2 p-1[/itex]. Unfortunately this isn't worth 1. What am I doing wrong?

Edit: I just found a mistake in my definition of Poisson's brackets. But now I find I) to be worth [itex]1+ \cot ^2 p-q \cot ^2 p[/itex] which is still wrong. I've rechecked the partial derivatives, I do not see any mistake, yet my result keeps being wrong.

Edit 2: This can be rewritten as [itex]\frac{1}{\sin ^2 p}-q \cot ^2 p[/itex]. This is not worth 1 so that the transformation isn't canonical, which is absurd. I do not see any mistake.

Edit 3 : I am nut guys! I found out the mistake in a derivative. I now reach all the results I should, problem solved!
 
Last edited:

FAQ: Show that a transformation is canonical

What is a canonical transformation?

A canonical transformation is a mathematical concept in classical mechanics that describes a change of coordinates that preserves the canonical form of Hamilton's equations. In simpler terms, it is a transformation that preserves the fundamental equations of motion of a physical system.

How do you show that a transformation is canonical?

To show that a transformation is canonical, one must demonstrate that it satisfies the conditions of the canonical transformation. These conditions include preserving the Hamiltonian, preserving the Poisson bracket, and being a one-to-one and onto mapping. If all of these conditions are met, the transformation is considered canonical.

What is the significance of canonical transformations?

Canonical transformations are significant in classical mechanics because they allow us to transform a set of equations into a different coordinate system without changing the physical behavior of the system. This can simplify calculations and provide different perspectives on the same physical system.

What are some examples of canonical transformations?

Examples of canonical transformations include shifts in position or momentum, rotations, and scaling transformations. Other examples can be found in specific physical systems, such as the Kepler problem or the harmonic oscillator.

Why are canonical transformations important in quantum mechanics?

In quantum mechanics, canonical transformations play a crucial role in the study of symmetries and conservation laws. They also help in the formulation of the Heisenberg uncertainty principle and the quantization of physical systems.

Back
Top